Art of Problem Solving

2025 AMC 10A Problems/Problem 2: Difference between revisions

Stevens0209 (talk | contribs)
Stressedpineapple (talk | contribs)
Tag: New redirect
 
(77 intermediate revisions by 26 users not shown)
Line 1: Line 1:
A box contains <imath>10</imath> pounds of a nut mix that is <imath>50</imath> percent peanuts, <imath>20</imath> percent cashews, and <imath>30</imath> percent almonds. A second nut mix containing <imath>20</imath> percent peanuts, <imath>40</imath> percent cashews, and <imath>40</imath> percent almonds is added to the box resulting in a new nut mix that is <imath>40</imath> percent peanuts. How many pounds of cashews are now in the box?
#redirect [[2025 AMC 12A Problems/Problem 2]]
 
<imath>\textbf{(A)}~3.5\qquad\textbf{(B)}~4\qquad\textbf{(C)}~4.5\qquad\textbf{(D)}~5\qquad\textbf{(E)}~6</imath>
 
==Solution 1==
 
We have <imath>5</imath> pounds of peanuts, <imath>2</imath> pounds of cashews, and <imath>3</imath> pounds of almonds in the first nut mix.
 
Let there be <imath>x</imath> pounds of nuts in the second nut mix, thus we have <imath>0.2x</imath> worth of peanuts. That means:
 
<imath>\(\frac{5 + 0.2x}{10 + x} = 0.4\)</imath>
 
<imath>5+0.2x=0.4(10+x)
5+0.2x=4+0.4x
1=0.2x
x=5</imath>
 
That means we have 5 pounds of the second nut mix. We are trying to find the amount of cashews in pounds.
 
<imath>10*0.2+5*0.4=2+2=4</imath>
 
 
There are a total of <cmath>\fbox{\textbf{(B)} 4}</cmath>

Latest revision as of 02:15, 8 November 2025